Show that R satisfies the radial Schrodinger equation

Then, after you plug in the function \psi(r,\theta,\phi)=R(r)Y(\theta,\phi), you'll have terms in both r and θ and ϕ. But since the equation is still true for any values of θ and ϕ, you can divide both sides by Y(\theta,\phi) and be left with just the part that depends on r. This is the equation you're trying to verify.In summary, to verify the given equation, one must plug in the function \psi(r,\theta,\phi)=R(r)Y(\theta,\phi) into the Schrödinger equation, express the Laplacian in spherical coordinates, and then separate the equation into two parts.
  • #1
warfreak131
188
0

Homework Statement



Starting with [tex]\psi(r,\theta,\phi)=R(r)Y(\theta,\phi)[/tex] saubstitute into the Schrodinger equation and show (using the technique of separation of variables) that R satisfies:

[tex](\frac{\hbar^2}{2m}\frac{1}{r^2}\frac{d}{dr}(r^2\frac{d}{dr})+\frac{C}{2mr^2}+V(r))R=ER(r)[/tex]

Homework Equations



[tex]L^2 Y(\theta,\phi)=CY(\theta,\phi)[/tex]
[tex]C=l(l+1)\hbar^2[/tex]

The Attempt at a Solution



The way I wrote it above is exactly the way the teacher wrote it, and I'm assuming that the R on the left side of the equation is not a function of r. And that being the case, it passes through the derivatives on the left hand side and and are left with a function of r.

C is a constant, so that entire term is also just a function of r. And V(r) itself is a function of r, therefore I'm inclined to think that you can pull out some constant E and multiply it by some function R(r) to give it the required format.
 
Physics news on Phys.org
  • #2
The R on the lefthand side is R(r).

The idea here is to plug the function [itex]\psi(r,\theta,\phi)=R(r)Y(\theta,\phi)[/itex] into the Schrödinger equation, expressing the Laplacian in spherical coordinates. After a little manipulation, you can separate the equation into two chunks, where one part depends only on r and the other depends only on θ and ϕ. The part that only depends on r is that equation you're trying to verify.
 
Last edited:
  • #3
vela said:
The R on the lefthand side is R(r).

The idea here is to plug the function [itex]\psi(r,\theta,\phi)=R(r)Y(\theta,\phi)[/itex] into the Schrödinger equation, expressing the Laplacian in spherical coordinates. After a little manipulation, you can separate the equation into two chunks, where one part depends only on r and the other depends only on θ and ϕ. The part that only depends on r is that equation you're trying to verify.

As the equation is right now, it has no theta or phi terms. Wouldnt I have to know the quantum numbers n, l, and m in order to introduce the other terms?
 
  • #4
You're supposed to start with the Schrödinger equation, which does have θ and ϕ terms.
 
  • #5
Therefore, I would say that R satisfies the radial Schrodinger equation.

However, to show this more rigorously, we can start by substituting the given expression for ψ into the Schrodinger equation:

[-(ħ^2/2m)(1/r^2)(d/dr)(r^2(d/dr)) + C/2mr^2 + V(r)]R(r)Y(θ, φ) = ER(r)Y(θ, φ)

Next, we use the technique of separation of variables by assuming that the wavefunction can be written as a product of two functions, one dependent only on r (R(r)) and the other dependent only on θ and φ (Y(θ, φ)). This allows us to separate the equation into two separate equations, one for R(r) and one for Y(θ, φ):

-[(ħ^2/2m)(1/r^2)(d/dr)(r^2(d/dr))R(r) + C/2mr^2R(r)]Y(θ, φ) + V(r)R(r)Y(θ, φ) = ER(r)Y(θ, φ)

Next, we divide both sides by R(r)Y(θ, φ) and rearrange the terms:

-[(ħ^2/2m)(1/r^2)(d/dr)(r^2(d/dr)) + C/2mr^2 + V(r)]R(r) = ER(r)

This is the same expression as the radial Schrodinger equation given in the problem statement, therefore R satisfies the equation. This shows that the radial part of the wavefunction, R(r), satisfies the radial Schrodinger equation.
 

1. What is the radial Schrodinger equation?

The radial Schrodinger equation is a mathematical equation that describes the behavior of a particle in a spherically symmetric potential. It is a key equation in quantum mechanics and is used to understand the behavior of subatomic particles.

2. How does R satisfy the radial Schrodinger equation?

In quantum mechanics, the wave function of a particle is denoted by the symbol Ψ. The radial Schrodinger equation is written in terms of the radial wave function R, which is a part of the overall wave function Ψ. R satisfies the equation by representing the radial component of the wave function and satisfying the conditions of the equation.

3. What are the conditions that R must satisfy in the radial Schrodinger equation?

There are several conditions that R must satisfy in order to satisfy the radial Schrodinger equation. These include being continuous, having a finite number of derivatives, and approaching zero as the distance from the center increases. Additionally, R must also satisfy the equation itself, which involves complex mathematical operations.

4. How is the radial Schrodinger equation used in practical applications?

The radial Schrodinger equation is used in a wide range of practical applications, particularly in the field of quantum mechanics. It is used to understand the behavior of electrons in atoms, as well as the properties of other subatomic particles. It is also used in the development of new technologies, such as quantum computers.

5. What are some limitations of the radial Schrodinger equation?

While the radial Schrodinger equation is a powerful tool in quantum mechanics, it does have some limitations. For example, it does not account for relativistic effects, which become important at very high speeds. It also does not take into account the effects of electromagnetic fields, which can have a significant impact on the behavior of particles. These limitations are addressed in more advanced equations, such as the Dirac equation.

Similar threads

  • Advanced Physics Homework Help
Replies
29
Views
142
  • Advanced Physics Homework Help
Replies
0
Views
463
  • Advanced Physics Homework Help
Replies
10
Views
2K
  • Advanced Physics Homework Help
Replies
1
Views
420
  • Advanced Physics Homework Help
Replies
4
Views
935
  • Advanced Physics Homework Help
Replies
10
Views
1K
  • Advanced Physics Homework Help
Replies
11
Views
2K
  • Advanced Physics Homework Help
Replies
3
Views
391
  • Advanced Physics Homework Help
Replies
30
Views
1K
  • Advanced Physics Homework Help
Replies
1
Views
1K
Back
Top